Applying similar logic but is not working: Prep 37 Forum

Prepare for the LSAT or discuss it with others in this forum.
Post Reply
eternallearner

Bronze
Posts: 101
Joined: Tue Dec 22, 2009 6:11 pm

Applying similar logic but is not working: Prep 37

Post by eternallearner » Sat Apr 03, 2010 1:05 am

In Prep 37 Section 2 Question 15, the correct choice is (B), but I picked (D). This time, my logic did not work for the question.

However, in Prep 37, Section 4 Question 20, I applied the same logic and it worked: (A) is the right answer.

Can someone explain why one logic approach worked in Question 20 and not in Question 15. I don't see the difference between these two type of LR?


On a separate issue, can you explain why Prep 37. Section 4. Question 17 has (E) as the correct choice. I picked (D) Had the question asked for a weakness in the argument, would (D) have been a correct choice?

Last question, why is Prep 37. Section 4. Question 21's answer (B)? What prompt in the question stem gave it away?

Thank you!

User avatar
KibblesAndVick

Silver
Posts: 533
Joined: Sun Feb 28, 2010 5:29 am

Re: Applying similar logic but is not working: Prep 37

Post by KibblesAndVick » Sat Apr 03, 2010 11:28 am

For Section 2 Question 15:
B is the correct answer because the conclusion states that we know more about the CRITIC than we do about the writer. The argument basically consists of reasons why we can't know what the writer meant (there are multiple ways to look at work and when people look at a work they are putting meaning into it not taking meaning away). What the argument has not yet done is establish why we would know something about the CRITIC. For all we know, the critics could be imposing ideas and meanings that have nothing to do with them as a person. Answer choice D simply adds another reason why we can't know jack about the author but fails to address the lack of information about the critic. If we can't know what the author meant, why are we able to know what the critics meant? Answer B answers this question.

For Section 4 Question 20:
Answer choice A is correct because it takes care of a "loophole" in the logic. The question never says that you can't be motivated by more than one thing. They try to throw you off by using the word "primarily" in the last sentence. But, just because your primary motivation is a desire to have everyone praise you, it doesn't mean that's your ONLY motivation. Maybe your decision was influenced by both a genuine desire to help others AND a genuine desire to be praised. Answer choice A closes this "loophole" by telling us that the two motivations are mutually exclusive.

I'm not sure what logic you used on both questions. If you can explain your thought process for the two questions in more detail maybe we can help you out a bit more.

For Section 4 Question 17:
E is TCR because if it were true it would undermine the argument. Therefore, for the argument to be a sound one, the author must assume it to be true (take it for granted). If the year before the speed limit reduction took effect there were an abnormally high number of car crashes (say for example the weather that year was terrible and there were a ton of car crashes involving hydroplaning and black ice) then we would expect to see a reduction in the number of car crashes REGARDLESS of what else happened. The fact that this years weather didn't suck as hard as last years weather did, BY ITSELF, would lower the number of car accidents. As such, if the logic is to follow that it was the reduction in the speed limit that is lowering the number of crashes we have to assume that the year before the change was a typical one.

Answer choice D could be a weakness in the argument. For example if the new speed limit was strictly enforced by a whole troop of officers then perhaps the lower number of accidents came about because there were so many damn cops on the road. Maybe if they had put that many cops on the road with the OLD speed limit, they would have seen the exact same result. However, as you seem to realize, the question is not asking for a weakness but rather an assumption. The author does not have to assume this to be true. It's one possibility that would undermine his argument but it is not an indispensable part of it.

For Section 4 Question 21:
Answer choice B is correct because it says "long term consequences are what matter most". This is what the prompt is also arguing. The theorist says that because the short term consequences are going to blow, we should not let the people repeal the law during this time of sucky-ness. Rather we should let things play out so people can see the long term gains in the law. But, why should we do that? It would be perfectly valid to argue that you don't want to have to deal with short term consequences that suck. It's annoying and disrupts your life. BUT.... if we assume principal B to be correct, this counterargument would fall flat. If what is truly important are the LONG TERM things and not the SHORT TERM then the theorist's argument in favor of forcing people to go through the short term suck for the long term gain makes a lot more sense.

Hope this helps and good luck kicking the LSAT's butt.

/Edited because me can't type English good/

Post Reply

Return to “LSAT Prep and Discussion Forum”